You are on page 1of 27

Physics - 2005

Prelim
1. A conducting spherical shell having inner radius a and outer radius b carries a net charge Q.
If a point charge q is placed at the centre of this shell, what is the surface charge density on
the outer surface ?
1) (Q - q)/(4b2)
2) (Q + q)/(4b2)
3) Q/(4b2)
4) (Q + q)/(4(b2 - a2))
2. On the T-S diagram, an isotherm is a straight line parallel to the entropy axis. What does the
area under the isotherm indicate?
1) The work done by the system
2) The change in internal energy of the system
3) The heat exchanged during the process
4) The work done on the system
3. A heat engine converts a given quantity of heat into work with maximum efficiency during
which one of the following processes?
1) Isobaric process
2) Isochoric process
3) Isentropic process
4) Isothermal process
4. When a wire is heated to a sufficiently high temperature, it is found to emit electrons. The
emitted electrons present in the interspace follow which one of the following distributions?
1) Maxwellian distribution
2) Boltzmann distribution
3) Lorentzian distribution
4) Exponential distribution
5. Consider the following statements in respect of van der Waals equation/van der Waals
forces :
1. The values of a and b in van der Waals equation do not vary with temperature.
2. Van der Waals forces are much weaker than those arising from valence bonds and are
inversely proportional to the seventh power of the distance between atoms or molecules.
Which of the statements given above is/are correct?
1) 1 only
2) 2 only

1/26

ias.edooni.com

3) Both l and 2
4) Neither 1 nor 2
6. On the inversion curve, the Joule-Thomson coefficient is
1) positive
2) zero
3) negative
4) infinite
7. For a particle of mass m in a one-dimensional potential
V(x) = (1/2)kx2 + ax
What is the ground state energy ?
{ = h/(2) and = (k/m)1/2}
1) /2
2) ( /2) + ak
3) ( /2) + (a2/2k)
4) ( /2) - (a2/2k)
8. If (1) represents isothermal and (2) represents adiabatic, which of the graphs given below in
respect of an ideal gas are correct ?

1) I and II
2) II and III
3) I and III
4) I, II and III
9. An ideal gas completes a cycle consisting of two isotherms and two isochors as shown in the
below figure. Which one of the following is correct ?

1) In the processes 1-2 and 2-3, entropy increases


2) In the processes 3-4 and 4-1, entropy increases
3) In the processes 1-2 and 4-1, entropy increases

2/26

ias.edooni.com

4) In the processes 2-3 and 3-4, entropy increases


10. What is the minimum attainable pressure of an ideal gas in the process given by T = a + bV2,
where a, b are constants and V is the volume of one mole of ideal gas ?
(R is the universal gas constant)
1) ab
2) Rab
3) 2Rab
4) (a/b)
11. How many degrees of freedom will the gas molecules have under standard condition
if is
the density and P is the pressure of the gas, and V is the velocity of sound propagating
through if ?
1) 2 [(V2 / P) - 1]
2) 2 [(V2 / P) - 1]-1
3) [(V2 / P) - 1]
4) [(V2 / P) - 1]-1
12. What is the quantity of heat required to raise the temperature of one gram molecule through
one degree for a monatomic gas at constant volume ?
1) 0.5 R
2) 1.0 R
3) 1.5 R
4) 2.5 R
13. van der Waals equation predicts that the critical coefficient of a gas (RT c/PcVc) has the value
1) 8/3

2) 2

3) 4/3

4) 3/8

14. If the most probable speed of a gas molecule of mass m is (2kT/m)1/2, where k is Boltzman
constant and T is the absolute temperature, then what will be the most probable value of
kinetic energy of the gas molecule ?
1) kT

2) 2kT

3) 3kT/2

4) kT/2

15. What is the relationship between pressure (p) and energy density () due to diffuse
radiation?
1) p = /3
2) p =
3) p = 2/3
4) p = /4
16. If heat Q is added reversibly to a system at temperature T and heat Q' is taken away from it
reversibly at temperature T', then which one of the following is correct ?
1) (Q/T) - (Q'/T') = 0

3/26

ias.edooni.com

2) (Q/T) - (Q'/T') > 0


3) (Q/T) - (Q'/T') < 0
4) (Q/T) - (Q'/T') = change in internal energy of the system
17. If a solid state power supply having voltage regulation 25% has full-load voltage of 20 V,
what will be its no-load voltage ?
1) 20 V
2) 25 V
3) 40 V
4) 50 V
18. Which one of the following elementary particles is called Baryon ?
1) Electron
2) -Meson
3) -Meson
4) Neutron
19. For the neutron decay process, n p + e + (y), where e is an electron, the particle (y) is a
1) neutrino
2) photon
3) pion
4) anti-neutrino
20. The binding energy per nucleon is largest for
1) 56Fe
2) 16O
3) 4He
4) 208Pb
21. The radius of Ge nucleus is measured to be twice the radius of 9Be4 nucleus. How many
nucleons are there in the Ge nucleus?
1) 72

2) 96

3) 120

4) 144

22. A particle of mass m and energy E is incident on a potential step of height V0 (at x = 0). If E <
V0, the wave function of the particle for x > 0 will be of the form
1) eikx
2) zero
3) e-kx
4) Aeikx + Be-ikx
23. If n represents the number of eigenstates of a hydrogen atom, then its discrete energy levels
are proportional to
4/26
ias.edooni.com

are proportional to
1) n
2) n2
3) 1/n
4) 1/n2
24. The eigenfunctions of hydrogen atom contain which of the following ?
1. Legendre polynomials
2. Laguerre polynomials
3. Hermite polynomials
Select the correct answer using the options given below :
1) 1, 2 and 3
2) 1 and 2
3) 1 only
4) 2 only
25. For a particle of m in a one-dimensional box of length l, what is the average of momentum px
for the ground state ?
1) 0
2) h/(2l)
3) h/l
4) h/(2l)
26. A particle constrained to move along the x-axis is described by the wve function
(x) = 2x; 0 < x < 1
= 0; elsewhere
What is the probability of finding the particle within the interval (0, 0.4) ?
1) 0.85
2) 0.085
3) 0.0085
4) 0.00085
27. Consider the following statements :
In order that measurable properties (such as energy, linear momentum, etc.) of a quantum
mechanical system by physically acceptable, it is essential that
1. the corresponding eigen-function(x) and its derivative (d
(x)/dx) must be finite for all
values of x.
2. (x) and (d(x)/dx) must be single valued for all values of x.
3. (x) and (d(x)/dx) must be continuous for all values of x.
Which of the statements given above are correct?
1) 1, 2 and 3
2) 1 and 2
3) 1 and 3
4) 2 and 3

5/26

ias.edooni.com

28. For the X-ray spectrum due to transition between n = 2 and n = 1 states, for large nuclear
charge Ze, we have frequencies v0, v1, v2 for Z 0 = Z 0, Z 1 = Z 0 + 1, Z 2 = Z 0 + 2 respectively.
Moseley's law implies which one of the following equations ?
1) v1 = (v0 + v2) / 2
2) v1 = (v0 v2) / 1/2
3) v11/2 = (v01/2 + v21/2) / 2
4) v1 = (v0 v2) / (v0 + v2)
29. A thermal neutron of rest mass m 0 has kinetic energy given by 3kT/2, where k is the
Boltzmann constant and T is the room temperature. What is the de Broglie wavelength
associated with such a neutron ?
1) h/(3m 0kT)
2) 3hkT/(2m 0)
3) 2h/(3kTm 0)
4) 2h/(3m 0kT)
30. In the Bragg scattering of a beam of electrons each of mass m and velocity v by a nickel
crystal, the first maximum is observed at = 30 ( being the angle the beam makes with
the crystal plane). What is the inter-planar distance d for the crystal?
1) h/mv
2) 2h/mv
3) h/2mv
4) mv/h
31. Two simple harmonic motions of same frequency but having a phase difference of travel at
right angles. When superimposed, what will be the nature of the resultant motion ?
1) A sine curve
2) A circle
3) An ellipse
4) A straight line
32. A cube at rest has a side of length 5m. What is the approximate volume of the cube, when
it moves with a velocity of 2 108 ms-1 parallel to one of its sides ?
1) 13.9 m 3
2) 11.2 m 3
3) 8.3 m 3
4) 6.7 m 3
33. For a spherical ball dropped in a long column of a viscous liquid, the speed (v) of the ball as
a function of time (t) may be best represented by which one of the curves shown below in
the graph?

6/26

ias.edooni.com

1) A

2) B

3) C

34. A unit mass at position vector


is moving with a velocity
the angular momentum of the body about the origin ?

4) D
. What is

1) 2 units along z-axis


2) 38 units along x-axis
3) 38 units along y-axis
4) 38 units along z-axis
35. A mass m attached to a spring is oscillating in water. If the spring constant is k and restoring
force is R , which one of the following is correct ?
1)
2)
3)
4)
36. A uniform spring of spring constant k is cust into two pieces whose lengths are in the ratio 1 :
2. What is the ratio of frequencies of oscillations in the vertical direction when mass m each
is suspended from the smaller and the larger pieces of the spring?
1) 1/2
2) 2
3) 1/2
4) 2
37. Which one of the following statements is correct ?
Binding energy of a nucleus is maximum when it contains
1) even number of protons and even number of neutrons
2) odd number of protons and odd number of neutrons
3) odd number of protons and even number of neutrons
4) even number of protons and odd number of neutrons
38. A projectile is projected with a speed u making an angle 2 with the horizontal. What is the
speed when its direction of motion makes an angle with the horizontal ?
1) (u cos 2)/2
2) u cos

7/26

ias.edooni.com

3) u (2 cos - sec )
4) u (cos - sec )
39. After cutting off a circular portion of radius R/2 from the centre of a uniform circular disc of
radius R, the moment of inertia about an axis passing through its centre and perpendicular
to its plane becomes I. What is the moment of inertia of the original disc about the same axis
?
1) 2I
2) 4I
3) (8/7) I
4) (16/15) I
40. If two bodies of mass M and 4M are attached to the two ends of a spring of spring constant
s, what is the period of oscillation?
1) 4(M/(5s))
2) 4(5s/M)
3) 2(5M/s)
4) 2(5M/(2s))
41. Which one of the following statements is correct ?
1) Gravitational force has an inverse-square dependence on distance, whereas electromagnetic, weak and strong forces are the short range forces
2) Gravitational and electromagnetic forces have the inverse-square dependence on
distance, whereas weak and strong forces are the short range forces
3) Gravitational electromagnetic and weak forces have the inverse-square dependence on
distance, whereas strong force is the short range force
4) Gravitational, electromagnetic, weak and strong forces have the inverse-square
dependence on distance
42. If the radius of earth were to decrease by 1%, its mass remaining the same, what would be
the approximate change in acceleration due to gravity ?
1) Decrease by 2%
2) Increase by 2%
3) Decrease by 1%
4) Increase by 1%
43. A gun shoots three times as high when its angle of elevation is as when it is . What is the
ratio of the horizontal distances travelled respectively in the two cases ?
1) 1 : 3
2) 1 : 1
3) 2 : 1
4) 3 : 5 : 1
44. A block B is resting on a horizontal plate in the x-y plane and the coefficient of friction

8/26

ias.edooni.com

between the black and the plate is . The plate begins to move in the x-direction and its
velocity is v = bt2, t being time and b being a constant. At what time will the block start sliding
on the plate ?
1) b/g
2) gb/2
3) g/b
4) g/2b
45. A body of mass M moves under the action of a central force with potential V (R) = AR3 (A >
0). For what kinetic energy will the orbit be a circle of radius r about the origin ?
1) 3AMr3/2
2) 3AMr/2
3) 3AMr4/2
4) 3AMr3
46. Moment of inertia of a uniform disc of mass m about an axis x = a is mk2, where k is the
radius of gyration. What is its moment of inertia about an axis x = a + b ?
1) mk2 + m (a + b)2
2) mk2 + m ((a + b)2/2)
3) mk2 + m (b2/2)
4) mk2 + mb2
47. A charge q is placed symmetrically with respect to three faces of equal area as shown in the
figure given below. What is the total electric flux through the faces ?

1) q/ 0
2) q/(2 0)
3) q/(3 0)
4) zero
48. Two parallel plate capacitors of capacitances C and 2C are connected in parallel and
charged to a potential difference V. The battery is then disconnected and the region between
the plates of the capacitor of capacitance C is completely filled with a material of dielectric
constant K. What is the potential difference across the capacitors now?
1) (3V)/(K + 2)

9/26

ias.edooni.com

2) (2V)/(K + 3)
3) KV
4) V/K
49. In an a.c. circuit with the combination of L and C, which one of the following represents the
variation of frequency (f) with reactance (X) ?
1)

2)

3)

4)

50. Which one of the following statements is correct ?


A material while superconducting behaves like
1) a diamagnet
2) a paramagnet
3) a ferromagnet
4) an antiferromagnet
51. Dielectric constant of mica is 6. What is the velocity of light in this medium approximately ?
1) 1.2 107m/s
2) 5.0 107m/s
3) 1.2 108 m/s
4) 3.0 108 m/s

10/26

ias.edooni.com

52. In a series resonant circuit, L = 1H, C = 0.25 f and the quality factor is 100. What is the
range of FWHM (Full Width at Half Maximum) of the resonant circuit ?
1) (1990 - 2010) rad / s
2) (1980 - 2020) rad /s
3) (1995 - 2005) rad / s
4) (1900 - 2010) rad / s
53. What is the kinetic energy gained by an electron due to acceleration through a potential
difference of 1 V?
1) 1 eV
2) 1 joule
3) 5 Nm
4) 10 Nm
54. Charge is distributed uniformly throughout a long non-conducting cylinder of radius R. Which
one of the following graphs best represents the magnitude of the resulting electric field E as
a function of r, the distance from the axis of the cylinder ?
1)

2)

3)

11/26

ias.edooni.com

4)

55. A loop of diameter d is rotated in a uniform electric field until the position of maximum electric
flux is found. The flux in this position is measured to be. What is the electric field strength
?
1) 4/d2
2) 2/d2
3) /d2
4) d2/4
56. A power supply P1 delivers 20 V d.c. with a ripple of 1.0 V r.m.s. while the power supply P2
delivers 50 V d.c. with a ripple of 2.0 mV r.m.s. Which one of the following is correct ?
1) P1 performs better than P2
2) P2 performs better than P1
3) Both P1 and P2 perform equally and similarly
4) Comparison between P1 and P2 is not possible
57. If the ratio of the concentration of electrons to that of holes in a semiconductor is 7/5 and the
ratio of the currents is 7/4, then what is the ratio of their drift velocities ?
1) 4/7

2) 5/8

3) 4/5

4) 5/4

58. In the figure below, Ec, Eg and Ev represent the conduction bandwidth, the band gap and the
valence of a semiconductor respectively. If the lattice constant of this semiconductor is
decreased, then which one of the following is correct ?

1) Ec, Ev and Eg increase


2) Ec, Ev and Eg decrease
3) Ec and Ev and increase but Eg decrease
4) Ec and Ev and decrease but Eg increase

12/26

ias.edooni.com

59. The circuit given below has two oppositely connected ideal diodes in parallel. What is the
current flowing in the circuit ?

1) 1.33 A
2) 1.71 A
3) 2.00 A
4) 2.31 A
60. A Zener diode has a breakdown voltage of 9.1 V with a maximum power dissipation of 364
mW. What is the maximum current that the diode can withstand ?
1) 0.04 A
2) 0.4 A
3) 4.0 A
4) 40 A
61. Which one of the following gives the output F for the logic diagram shown below ?

1)
2)
3)
4)
62. The number of bits that a digital computer can process in parallel at a time is called
1) Byte
2) Binary digit
3) Word length
4) PACE
63. If the memory chip size is 256 1 bits, what is the number of chips required to make up 1
kbyte of memory ?
1) 32 chips
2) 64 chips
3) 128 chips

13/26

ias.edooni.com

4) 256 chips
64. Given below are four types of read only memories used in microcomputers :
1. EPROM
2. PROM
3. ROM
4. EAROM
Which one of the following is the correct sequence in terms of maximum to minimum ease
for storing program in them ?
1) 2 - 3 - 1 - 4
2) 4 - 1 - 2 - 3
3) 3 - 2 - 1 - 4
4) 4 - 1 - 3 - 2
65. A radioactive nucleus has a half-life of 100 years. If the number of nuclei at t = 0 is N0, what
will be the number of nuclei which have decayed in 300 years ?
1) 7 N0 / 8
2) N0 / 2
3) 3 N0 / 4
4) N0 / 8
66. A beam of atoms splits into 4 components in the Stern-Gerlach experiment with a weak
magnetic field. What is the total angular momentum quantum number j for these atoms ?
1) 3/2

2) 1/2

3) 3

4) 4

Directions for question 67 to 72 :


The following consist of two statements : one labelled as the Assertion (A) and the
other as Reason (R). You are to examine these two statements carefully and select
the answers to these items using the options given below :
(a) Both A and R are individually true, and R is the correct explanation of A
(b) Both A and R are individually true but R is not the correct explanation of A
(c) A is true but R is false
(d) A is false but R is true
67. Assertion (A) : Energy is released when two light elements combine to form a heavier
element.
Reason (R) : Binding energy per nucleon decreases with mass number in the region of
large mass numbers.
1) a

2) b

3) c

4) d

68. Assertion (A) : A disc rolls down on inclined plane faster than a ring of same outer radius,
material and thickness.
Reason (R) : Radius of gyration of disc is greater than that of ring.
1) a

2) b

3) c

14/26

4) d

ias.edooni.com

69. Assertion (A) : The energy eigenvalue of the lowest energy state of a simple harmonic
oscillator is called zero-point energy.
Reason (R) : Energy of this state is zero.
1) a

2) b

3) c

4) d

70. Assertion (A) : In quantum mechanics, a particle with total energy E can tunnel through a
potential barrier of height V0 greater than E, which is classically impenetrable due to kinetic
energy becoming negative inside the barrier.
Reason (R) : Wave particle duality in quantum mechanics allows the particle to have
negative kinetic energy.
1) a

2) b

3) c

4) d

71. Assertion (A) : It is possible to obtain Newton's rings with transmitted light.
Reason (R) : Phase change of is introduced in any one beam of the transmitted light.
1) a

2) b

3) c

4) d

72. Assertion (A) : In almost all measuring optical instruments, Ramsden eyepiece is used.
Reason (R) : A cross wire or a scale can be fitted at the position of the image formed by the
objective.
1) a

2) b

3) c

4) d

73. A spring oscillating in water is acted upon by an external force B cos t. With the passage of
time, the frequency of the spring tends to be
1) greater than
2) less than
3) equal to
4) decreasing exponentially
74. Reference frame A is moving with a velocity VA with respect to earth and reference fram B is
moving with a velocity VB with respect to reference frame A. If M A and M B are the masses of
a body in A and B respectively and c is the speed of light, then what is the value of M A/M B ?
1) (1 V2B/c2)
2) [1 ((VA VB)2/c2)]-1/2
3) (1 (VA VB)2/c2)
4) (1 V2B/c2)-1/2
75. A particle of rest mass (10m)/3 decays at rest into rwo mesons of mass m each. What is the
velocity of each meson in terms of the velocity of light c ?
1) 3c/10
2) 3c/5
3) 2c/5
4) 4c/5

15/26

ias.edooni.com

76. A cube has density 0 when at rest. What is the density of the cube when it moves with
velocity v (close to c) parallel to one of its sides ?
1) 0 (1 - v2/c2)1/2
2) 0 (1 - v2/c2)-1/2
3) 0 (1 - v2/c2)-1
4) 0 (1 - v2/c2)
77. Which one of the following represents relativistic version of Newton's Second Law of motion
with rest mass m 0 and speed v ?
1) F = m 0 (dv/dt)(1 - v2/c2)-3/2
2) F = m 0 (dv/dt)(1 - v2/c2)3/2
3) F = m 0 (dv/dt)(1 - v2/c2)-1/2
4) F = m 0 (dv/dt)(1 - v2/c2)
78. A source of sound approaches an observer and then recedes from it. Ratio of frequencies of
sound as the source approaches and as the source recedes is 6 : 5 What is the speed at
which the source is moving ?
(Velocity of sound = 330 m s-1)
1) 24 m s-1
2) 27 m s-1
3) 30 m s-1
4) 33 m s-1
79. A light ray is travelling through a ring of an optical fibre which is made of four different
glasses (shown above) but each part has the same geometrical thickness, if their refractive
indices are (i) 1.51, (ii) 1.52, (iii) 1.53, (iv) 1.54 respectively, light ray will take the maximum
time in crossing which part ?

1) Part No. 1
2) Part No. 2
3) Part No. 3
4) Part No. 4
80. A convex lens of optical glass ( = 1.42) is immersed in a liquid ( = 1.49) inside a glass cell.
How does the lens-liquid combination act ?

16/26

ias.edooni.com

1) As a converging lens
2) As a diverging lens
3) As a piano-parallel lens
4) As a piano-convex lens
81. Let F R and F V be the focal length corresponding to red and violet colours respectively for a
combination of lenses. For removal of chromatic aberration, which one of the following
conditions holds good ?
1) (F R/F V) = 1
2) (F R/F V) = 2
3) (F R/F V) > 1, but not equal to 2
4) (F R/F V) < 1
82. If the refractive index of the material of a prism is cot (A/2) and vertex angle of the prism is A,
what is the angle of minimum deviation ?
1) - 2A
2) - A
3) (/2) - 2A
4) (/2) - A
83. Light takes 4.5 10-10 s to travel 10 cm in a transparent medium. What is the optical path
length covered by it ?
1) 11.5 cm
2) 13.5 cm
3) 15.5 cm
4) 17.5 cm
84. Which one of the following statements is correct ?
A zone plate is used as a convex lens to focus light of wavelength . The focal length is
1) independent of in the visible region
2) proportional of in the visible region
3) inversely proportional to and is more for violet rays than for red rays
4) proportional to -1/2 and is more for violet rays than for red rays
85. A circularly polarised beam of light passes through a quarter wave plate. The emerging
beam is
1) plane polarised
2) circularly polarised
3) elliptically polarised
4) partially polarised
86. When an unpolarised light beam passes through a double refracting medium, it splits up into

17/26

ias.edooni.com

two beams called ordinary ray and extraordinary ray.


Consider the following statements :
1. Intensities of the both rays are equal.
2. Refractive index of the ordinary ray remains constant.
3. Refractive index of the extraordinary ray does not remain constant.
4. Both the rays are unpolarised.
Which of the statements given above are correct ?
1) 1, 2 and 3
2) 1 and 4
3) 2 and 3
4) 3 and 4
87. Which one of the following statements is correct ?
Optically active substances are responsible for
1) the rotation of the plane of polarisation of polarised light
2) producing polarised light
3) producing bifringence
4) converting ordinary light into polarised light
88. The coefficient of thermal conductivity of a gas is proportional to
1) T 2
2) T
3) 1/T
4) T
89. If the mean kinetic energy of a smoke particle executing Brownian motion in air is Esmoke
and the mean kinetic energy of an air molecule is Eair, then which one of the following is
correct?
1) Esmoke > Eair
2) Esmoke = Eair
3) Esmoke < Eair
4) No relation between Esmoke and Eair can be established
90. The equation of state of a gas is given as
P (V - b) = nRT
where b is a constant, n is the number of moles and R is the universal gas constant. When 2
moles of this gas undergo reversible isothermal expansion from volume V to volume 2V,
what is the work done by the gas ?
1) 2RT ln [(V - b)/(2V - b)]
2) 2RT ln [(2V - b)/(V - b)]
3) 2RT ln [(V - b)/(2V)]
4) 2RT ln [(2V)/(V - b)]
91. Consider the following statements :

18/26

ias.edooni.com

Some of the important results of Andrews' experiment are that


1. there exists a temperature called critical temperature, above which a gas cannot be
liquefied however great the applied pressure is.
2. oxygen, nitrogen and hydrogen are permanent gases and they cannot be liquefied.
Which of the statements given above is/are correct ?
1) 1 only
2) 2 only
3) Both 1 and 2
4) Neither 1 nor 2
92. The plots of Maxwell's distribution function (dN/dc) versus speed (c) for a given sample of a
gas at three different temperatures T 1, T 2 and T 3 respectively are shown in the below
diagram. If the areas on the c-axis under three curves I, II and III be denoted by AI, AII and
AIII respectively, then which one of the following is correct ?

1) AI > AII > AIII


2) AI = AII = AIII
3) AI < AII < AIII
4) AII < AI < AIII
93. A quarter wave plate is placed over a shiny coin. A plane polariser is placed on top of the
quarter wave plate such that the transmission axis of the polariser is at 45 to the fast axis of
the quarter wave plate. How does the shiny coin appear now ?
1) Dark
2) Shiny as before
3) Shinier than before
4) Coloured
94. What is the ground state energy of positronium ? (The ground state energy of hydrogen is 13.6 eV)
1) -3.4 eV
2) -6.8 eV
3) -13.6 eV
4) -27.2 eV

19/26

ias.edooni.com

95. Match List-I (Different Types of Laser) with List-II (Active Medium) and select the correct
answer using the options given below the lists :
List-I
List-II
(Different Types of Laser)
(Active Medium)
A

Ruby Laser

Liquid laser

He-Ne Laser

Solid state laser

Dye Laser

Gas laser

Injection Laser

Junction semiconductor laser

1) A-2

B-1

C-3

D-4

2) A-4

B-3

C-1

D-2

3) A-2

B-3

C-1

D-4

4) A-4

B-1

C-3

D-2

96. A soap film of thickness d and refractive index is illuminated by white light incident at an
a n g le i. The light refracted at an angle
is examined by a spectroscope, and two
consecutive bright bands are seen corresponding to wafelengths

1 and 2 . The path


difference 2D cos is given by
1) (1 - 2)/(1 2)
2) (1 + 2)/(1 2)
3) (1 2)/(1 + 2)
4) (1 2)/(1 - 2)
97. In a diffraction grating, slit width is e and ruling separation is d. If second-order spectrum is
to be avoided, one should choose
1) e = d/2
2) e = 2d
3) e = 2d
4) e = d
98. A diffraction grating has N lines and grating element is (e + d). For wavelength , what is the
maximum resolving power possible?
1) N (e + d)/
2) (e + d)/(N)
3) (e + d)N2/
4) (e + d - )2N/2
99. The core of an optical fibre has refractive index 1.6 and its cladding has refractive index 1.5.
What is the approximate numerical aperture of the optical fibre ?
1) 0.31

2) 0.56

3) 0.68

4) 0.88

100. Match List-I (Phase Difference between Two Similar Superimposed Waves having Mutually
Perpendicular States of Polarisation and Propagating through the Same Axis) with List-II

20/26

ias.edooni.com

(Results) and select the correct answer using the options given below the lists :
List-I
(Phase Difference between Two Similar Superimposed Waves
List-II
having Mutually Perpendicular States of Polarisation and
(Result)
Propagating through the Same Axis)
A

= /2

= 2/3

= 3/2

1) A-1

B-3

C-4

D-2

2) A-4

B-2

C-1

D-3

3) A-1

B-2

C-4

D-3

4) A-4

B-3

C-1

D-2

Linearly
1 polarised
light
Left
circularly
2
polarised
light
Right
circularly
3
polarised
light
Elliptically
4 polarised
light

101. Match List-I (Optical Instrument) with List-II (Characteristics of the Instrument) and select
the correct answer using the options given below the lists :
List-I
(Optical
Instrument)
A

Huygens eyepiece

List-II
(Characteristics of the Instrument)
1

Analysis of elliptically polarised light

B Optical fibre

Chromatic and spherical aberrations are reduced to a minimum

Babinet's
compensator

Production of circularly polarised light

Quarter wave
plate

Refractive index is maximum at the axis and decreases


towards the periphery

1) A-2

B-4

C-1

D-3

2) A-3

B-4

C-1

D-2

3) A-2

B-1

C-4

D-3

4) A-3

B-1

C-4

D-2

102. Two light waves having their intensities in the ratio 16 : 9 interfere to produce interference
pattern. What is the ratio of maximum intensity to minimum intensity in this pattern ?
1) 4 : 3

21/26

ias.edooni.com

2) 25 : 7
3) 625 : 49
4) 49 : 1
103. Consider the following statements :
The continuous X-rays spectrum is the result of
1. inverse photo-electric emission.
2. decrease in the kinetic energy of the incident electrons on the target atom.
3. annihilation of the mass of the incident electron.
Which of the statements given above are correct ?
1) 1, 2 and 3
2) 1 and 2
3) 2 and 3
4) 1 and 3
104. A -meson which is about 200 times as heavy as an electron is captured by a proton to
form a hydrogen-like atom. What is the radius of the first Bohr orbit ? (The radius of first
Bohr orbit for normal hydrogen atom is 0.5 )
1) 100
2) 0.25
3) 0.025
4) 0.0025
105. Match List-I with List-II and select the correct answer using the options given below the lists
:
List-I

List-II

Goudsmit and Uhlenbeck

Antisymmetric state

Moseley's plot

Space quantization

Pauli exclusion

Bohr's theory

Stern-Gerlach

Electron spin

1) A-1

B-3

C-4

D-2

2) A-4

B-3

C-1

D-2

3) A-1

B-2

C-4

D-3

4) A-4

B-2

C-1

D-3

106. For what value of g is the function x + gy, an eigenfunction of Lz with eigenvalue - , where
, where = (h/2), h being the Planck's constant ?
1) g = 1
2) g = -1
3) g = i
4) g = -i

22/26

ias.edooni.com

107. The frequency of the radiation emitted by a hydrogen atom for the transition between n = 2
and n = 1 states is v0. What is the frequency of the radiation emitted by the hydrogen atom
for transition between n = 4 and n = 1 states ?
1) 3v0/2
2) 2v0
3) 4v0
4) 5v0/4
108. The maximum energy of deuterons coming out of a cyclotron accelerator is 20 MeV. What
is the maximum energy of protons that can be obtained from the accelerator ?
1) 10 MeV
2) 20 MeV
3) 30 MeV
4) 40 MeV
109. Which one of the following relations describes the variation of refractive index () of an
optical substance with wavelength () of light ? (Neglect higher order terms)
1) = A + B 2
2) = A + B -1
3) = A + B -2
4) = A + B
110. Which one of the following circuits is electrically equivalent to the circuit given below ?

1)

2)

23/26

ias.edooni.com

3)

4)

111. In a Hall effect experiment, what happens to Hall voltage if the applied magnetic field is
doubled, keeping the Ohmic current density unchanged ?
1) It increases 4 times
2) It becomes half
3) It remains unchanged
4) It increases 2 times
112. A circular loop of resistance R and encompassing an area A is subjected to a magnetic field
B0 e-bt perpendicular to its plane, B0 and b being constants, and t is the time. What is the
induced current at time t = 0?
1) B0bA/R
2) B0bR/A
3) B0bR/b
4) B0bR/2A
113. What is the Poynting vector at the surface of a long cylindrical wire of radius R, length L
carrying a current I, when its ends are kept at a potential difference of V?
1) zero
2) VI/(2R2 + 2RL)
3) VI/(R2L)
4) VI/(2RL)
114. Black-body radiation behaves like a perfect gas of adiabatic exponent () equal to
1) 1/2

2) 1/3

3) 1

4) 4/3

115. If the value of van der Waals constant b for a real gas is 32 cm 3/mole, then what is the
approximate volume of one molecule of the gas ? (Avogadro constant = 6.02 1023)
1) 5.2 10-23 cm 3
2) 3.9 10-23 cm 3
3) 2.6 10-23 cm 3
4) 1.3 10-23 cm 3

24/26

ias.edooni.com

116. What is the value of current passing through the branch AB in the electric circuit given below
?

1) 8/7 A
2) 2/3 A
3) 3/2 A
4) 3/7 A
117. Which one of the following graphs represents the variation of magnetic flux density (B) with
distance (r) from a long straight conductor carrying current ?
1)

2)

3)

4)

118. What is the approximate peak value of an alternating current producing four times the heat
produced per second by a steady current of 2.0 A in a resistor ?

25/26

ias.edooni.com

1) 2.8 A
2) 4.0 A
3) 5.6 A
4) 8.0 A
119. Which one of the following statements is correct ?
If in the Wheatstone bridge, the battery and galvanometer are interchanged, the condition
for balance
1) is not disturbed
2) is disturbed
3) depends on internal resistance of the battery
4) depends of the values of resistances in the bridge
120. Consider a rhombus ABCD with B = 120. A charge q placed at corner A prodces field E
and potential V at the corner D. If we now add charges -2q and +q at the corners B and C
respectively, what will be magnitudes of the field and potential at D respectively ?
1) E and 0
2) 0 and V
3) (2) E and V/2
4) E/(2) and V/2

26/26

ias.edooni.com

Answer Key
1) 2

2) 1

3) 3

4) 2

5) 2

6) 2

7) 1

8) 4

9) 3

10) 3

11) 1

12) 1

13) 1

14) 4

15) 1

16) 1

17) 2

18) 4

19) 4

20) 1

21) 1

22) 3

23) 4

24) 2

25) 2

26) 2

27) 1

28) 3

29) 1

30) 3

31) 1

32) 3

33) 2

34) 1

35) 4

36) 1

37) 1

38) 3

39) 3

40) 1

41) 2

42) 2

43) 2

44) 4

45) 1

46) 4

47) 2

48) 1

49) 4

50) 1

51) 3

52) 1

53) 1

54) 1

55) 1

56) 2

57) 4

58) 3

59) 3

60) 1

61) 1

62) 3

63) 1

64) 2

65) 1

66) 3

67) 1

68) 1

69) 3

70) 1

71) 1

72) 1

73) 3

74) 1

75) 4

76) 2

77) 1

78) 3

79) 4

80) 2

81) 1

82) 1

83) 2

84) 3

85) 1

86) 3

87) 1

88) 4

89) 2

90) 2

91) 1

92) 2

93) 4

94) 2

95) 3

96) 4

97) 4

98) 1

99) 2

100) 1

101) 4

102) 4

103) 1

104) 4

105) 2

106) 1

107) 4

108) 1

109) 3

110) 1

111) 4

112) 1

113) 4

114) 4

115) 4

116) 1

117) 3

118) 2

119) 1

120) 1

1/1

ias.edooni.com

You might also like